Trash-A-Way charges $25 per month with an additional $0.25 per pound of trash that they pick up from your house. Garbage-No-More charges a flat fee of $45 per month. Write a system of equations to represent each plan. How many pounds of trash would make the two plans equal?

Answers

Answer 1

Answer:

look at the explanation

Step-by-step explanation:

Write an equation:

Trash a way : $25 + 0.25x

Garbage No More : $45 per month

so to make them equal we need to have enough trash in Trash A Way that it equals $45

25  + 0.25x =45

0.25x = 20

x = 80

So, you need to have 80 pounds of trash to make them equal

hope this helps!

please vote and heart!


Related Questions

Please help me out with this one!

Answers

Are you sure you need this it’s summer

Answer:

87 in^2

Step-by-step explanation:

24 + 18 + 15 + 30

the 30  is 2(1/2 * 5 * 6)

f(x) = x2 + 3x + 2 is shifted 2 units left. The result is g(x). What is g(x)?

Answers

Answer:

[tex]g(x) = x^2+7x+12[/tex]

Step-by-step explanation:

If we are given a function f and we want to shift it a units horizontally, the resulting function will be f(x - a), where a positive a is a shift rightwards and a negative a is a shift leftwards.

We have the function:

[tex]f(x)=x^2+3x+2[/tex]

And we want to shift it two units to the left.

Since we want to shift it two units to the left, a = -2. Therefore, our new function, let's call it g, must be f(x - (-2)) or f(x + 2). Substitute:

[tex]g(x) = f(x+2)=(x+2)^2+3(x+2)+2[/tex]

Simplify. Expand:

[tex]g(x) = (x^2+4x+4)+(3x+6)+2[/tex]

Combine like terms. Hence:

[tex]g(x) = x^2+7x+12[/tex]

aint the answer for this 10 just let me if im wrong​

Answers

Answer:

Yup

Step-by-step explanation:

please I really need help with this

Answers

Answer:

24

Step-by-step explanation:

(1+25) - (16)(1/8)

26 - 2

24

Cheers

Answer:

24

Step-by-step explanation:

(1+5^2) -16 ( 1/2)^3

PEMDAS

Parentheses first

Exponents in the parentheses

(1+25) -16 ( 1/2)^3

Then add in the parentheses

(26) -16 ( 1/2)^3

Then exponents

(26) -16 ( 1/8)

Then multiply

26 - 2

Then subtract

24

What is the measure of n

Answers

Answer:

n = [tex]\sqrt{32}[/tex]

Step-by-step explanation:

The 2 right triangles inside the larger right triangle are similar , so corresponding sides are in proportion

[tex]\frac{8}{n}[/tex] = [tex]\frac{n}{4}[/tex] ( cross- multiply )

n² = 32 ( take the square root of both sides )

n = [tex]\sqrt{32}[/tex]

What is the value of x that makes the given equation true?
2(x−8)=x+5x

a x=-4
b x=-16
c x=-2
d x=0

Answers

Answer:

x = -4

Step-by-step explanation:

2(x−8)=x+5x

Distribute

2x - 16 = x+5x

Combine like terms

2x-16 = 6x

Subtract 2x from each side

2x-16 = 6x-2x

-16 = 4x

Divide by 4

-16/4 = 4x/4

-4 =x

[tex]\boxed{\large{\bold{\textbf{\textsf{{\color{blue}{Answer}}}}}}:)}[/tex]

[tex]\sf{2(x-8)=x+5x }[/tex]

[tex]\sf{2x-16=6x }[/tex]

[tex]\sf{2x-6x=16 }[/tex]

[tex]\sf{-4x=16 }[/tex]

[tex]\sf{ x=\dfrac{16}{-4} }[/tex]

[tex]\sf{x=-4 }[/tex]

[tex]\sf{ }[/tex]

HELP QUICK PLEASE!

Given: AC BD and AD BC
B
D
What postulate could prove DCA ACDB
O Side-Side-Side
O Side-Angle-Side
Angle-Side-Angle
Angle-Angle-Side

Answers

Answer:

Step-by-step explanation:

Side - side - side. In fact the two triangles have the side CD in common

An experiment consists of rolling a six sided die to select a number between between 1 and 6 and drawing a card at random from a set of 10 cards numbered 1,2,3,...10. Which event definition corresponds to exactly one outcome of the experiment?

Answers

Answer:

Man this is confusing

Step-by-step explanation:

Find the value of x.
A. 70
B. 30
C. 65
D. 40

Answers

Answer:

Option (A)

Step-by-step explanation:

From the figure attached,

By applying triangle sum theorem in ΔBCP,

m∠CBP + m∠CPB + m∠BCP = 180°

15° + 20° + m∠BCP = 180°

m∠BCP = 145°

m∠ACB + m∠BCP = 180° [Linear pair of angles are supplementary]

m∠ACB + 145° = 180°

m∠ACB = 35°

Since, ∠ACB is the inscribed angle and AB is the intercepted arc.

Therefore, m(arc AB) = 2m(∠ACB)

m(arc AB) = 2 × 35°

                 = 70°

Therefore, Option (A) is the correct option.

add 7 with the diffrence of 12 and 9 =​

Answers

Answer:

7+3 = 10

Step-by-step explanation:

if I understand the problem strangeness right, and there is nothing missing in it, then this is the simple answer.

the difference of 12 and 9 is 12-9 = 3.

and then adding this 3 to 7 is 7+3 =10

Answer:

10

Step-by-step explanation:

The difference of 12 and 9 is 3.

Now,

If we add 7 to 3, the answer is 10.

C=-(251x3+281)+3X251-(1-281)

Answers

Answer:

-1

Step-by-step explanation:

=-251x3+281 +251x3-1+281

=-1

Can someone help me with this math homework please!

Answers

Answer:

The graph for a member's yearly cost for total game tokens purchased would have a y-intercept of (0, 60). However, the nonmember's yearly cost would have a different y-intercept of (0,0). This is because the member's yearly cost has a yearly initial fee of $60, while as the nonmember yearly cost doesn't include an inital fee. Also, the graph for the nonmember's yearly cost would be steeper than the member's graph because the slope 1/5 is greater than 1/10.

Hope this helps (●'◡'●)

The fractions 1/10 and 1/5 convert to 0.1 and 0.2 respectively.

So the equations y = (1/10)x+60 and y = (1/5)x are the same as y = 0.1x+60 and y = 0.2x in that order.

We see that the nonmember cost graph is steeper since the slope 0.2 is larger than 0.1; the further we move away from 0, the steeper the slope. A steeper slope means that you pay more per token. This is the unit cost.

Specifically, you would pay 0.20-0.10 = 0.10 more dollars per token if you are a nonmember compared to if you are a member.

---------------------------

Also, notice that the y intercepts of y = 0.1x+60 and y = 0.2x are 60 and 0 in that order. The members pay an up front fee of $60 before they can buy any tokens. Nonmembers don't have to pay an upfront cost.

Visually, the graph for the members will be much higher up compared to the nonmembers graph. Both lines are increasing, but the members graph is increasing less overall. At some point, the two lines intersect. After this point, it makes more sense to be a member.

What is the volume of the cylinder below?
A. 196pi units3
B. 784pi units3
C. 98pi units 3
D. 112pi units 3

Answers

Answer:

196π units^3

Step-by-step explanation:

The volume of a cylinder is:

πr^2h

SINCE, there is no given pi, the formula will be:

r^2h

Use the formula on the given dimensions:

r^2h

= (7^2) 4

= (49) 4

= 196

So we could just leave the final answer as:

196π

The real volume will be solved once π is given.

I hope I helped!

The volume of the cylinder given is 196 π, Option  A is correct.

The volume of the cylinder is given by V=πr²h

Where r is radius of cylinder and h is the height of the cylinder.

V is the volume

r=7

h=4

Now plug in these values in volume formula.

V=π(7)².4

=π.49.4

V=196π

To learn more on Volume click:

https://brainly.com/question/13798973

#SPJ7

The length of a boat in scale drawing is 8 in. Find the length of the actual
boat if the scale is 1 in = 3 ft.
A)24 in
B.)24 ft
C.) 8 ft
D.) 11 in.

Answers

Answer:

B. 24 ft

Step-by-step explanation:

1 in = 3ft

8 in must equal 24 ft since 8 in x 3 ft is 24 ft. So, B.

The length of the boat will be equal to 24 ft.

What is the scale factor?

The scale factor is defined as the proportion of the new image's size to that of the previous image. decision-making.

Given that:-

The length of a boat in scale drawing is 8 in. Find the length of the actual boat if the scale is 1 in = 3 ft.

The actual length will be calculated as follows:-

1 in = 3 ft.

8 in = 8 x 3 ft

8 in = 24 ft.

Therefore the length of the boat will be equal to 24 ft.

To know more about scale factors follow

https://brainly.com/question/25722260

#SPJ2

need help asap pleaseeeeeeeeeeeeeeeeeeeeeeeeeee
PLEASEEEEEEEEEEEEEEEEEEEEEEEEEEEEEEEEEEEEEEEEEEE

Answers

9514 1404 393

Answer:

  4a. 177

  4b. 248

  5a. 27

  5b. 9

  5c. 12

Step-by-step explanation:

4.

a) x is 3/4 of 1/2 of all, so is 3/8 of all.

  x = 3/8·472

  x = 177

b) x is 2/5 of 2/3 of all, so is 4/15 of all.

  x = 4/15·930

  x = 248

__

5.

Elaine gave her grandmother 3/4 of 48 = (3/4)(48) = 36 of the flowers. Then Elaine had 48 -36 = 12 flowers left.

Her grandmother gave 1/4 of 36 flowers = (1/4)(36) = 9 of the flowers to her neighbor. Elaine's grandmother has 36 -9 = 27 flowers now.

Elaine's grandmother has 27 flowersthe neighbor has 9 flowersElaine has 12 flowers

PLEASE ANSWERRRRR UR SO COOL IF YOU DOOOO PLSSSS

Answers

Answer:

3x-7y and -7y+ 3x

Step-by-step explanation:

3x +  (-7y) = (-7y) + 3x

Answer:

Step-by-step explanation:

The first set of expressions are equivalent; they are identical except for order of the terms.

Select the relationship that does represent a function.

Answers

Answer:

c

Step-by-step explanation:

Which of the following could be the first step in solving the equation below ? 5 ^ x = 21

Answers

Answer:

D

Step-by-step explanation:

usually when your variable is an exponent you always take the log of both sides.

The possible first step of solving the equation is log 5^x = log 21

How to determine the first step?

The equation is given as:

5^x = 21

Take the logarithm of both sides of the equation

log 5^x = log 21

The above represents the possible first step of solving the equation

Hence, the possible first step of solving the equation is log 5^x = log 21

Read more about equations at:

https://brainly.com/question/2972832

(6x^2-4x-5)(2x^2+3x)

Answers

Answer:

[tex]\left(6x^2-4x-5\right)\left(2x^2+3x\right)[/tex]

[tex]=6x^2\cdot \:2x^2+6x^2\cdot \:3x+\left(-4x\right)\cdot \:2x^2+\left(-4x\right)\cdot \:3x+\left(-5\right)\cdot \:2x^2+\left(-5\right)\cdot \:3x[/tex]

[tex]=6\cdot \:2x^2x^2+6\cdot \:3x^2x-4\cdot \:2x^2x-4\cdot \:3xx-5\cdot \:2x^2-5\cdot \:3x[/tex]

[tex]=12x^4+10x^3-22x^2-15x[/tex]

OAmalOHopeO

Carol wants to build a fence around her
garden to prevent rabbits from eating the
plants. The fencing costs $15 per metre.
How much will it cost to enclose a square
garden with an area of 81 m2?
(Hint: Area of a square = 5?, perimeter of a
square = 4s)

Answers

Answer:

540$

Step-by-step explanation:

9x9 = 81

9 + 9 + 9 + 9 = 36

36 x 15 =540

can i get some help. did it 3 times and still not sure.

Answers

Answer:

18

Step-by-step explanation:

The height of the cone is 8, and the diameter is 3. This means that the radius is 3/2, or 1.5. We can plug everything into the formula (using 3 for pi):

[tex]\frac{3(1.5)^2(8)}{3}[/tex]

We can cancel out the 3 from the numerator and the denominator, so we are left with 1.5^2 times 8. 1.5 squared is equal to 2.25, and 2.25 times 8 is equal to 18.

1. 80 = -10b
2. 6 = 2n
3. -16r = 32

1.
2.
3.

Answers

Answer:

1. - 8

2. 3

3. - 2

Step-by-step explanation:

1.

80 = - 10b

- 10b = 80

b = 80 / - 10

b = - 8

2.

6 = 2n

2n = 6

n = 6 / 2

n = 3

3.

- 16r = 32

r = 32 / - 16

r = - 2

Answer:

1. b = - 8
2. n = 3
3. r = -16

Step-by-step explanation:

we’re trying to solve for each variable

1. 80 = -10b

Divide both sides by -10

80/-10 = -10b/-10

-8 = b

2. 6 = 2n

Divide both sides by 2

6/2 = 2/2n

3 = n

3. -16r = 32

Divide -16 by both sides

-16r/-16 = 32/-16

r = -16

Does anyone know how to do these?

Answers

Answer:

D.

Step-by-step explanation:

f(x) = [tex]\sqrt[3]{4x}[/tex]

g(x) = 2x + 3

[tex]\frac{f}{g}[/tex] (x)  = 'f' divided by 'g'

[tex]\frac{\sqrt[3]{4x} }{2x+3}[/tex]    Substitute.

x cannot be -[tex]\frac{3}{2}[/tex] because that would create a zero in the denominator.

That's a no-no.

It makes it 'undefined'

Please help me with this.. and tell correct answer..​

Answers

Answer:

Step-by-step explanation:

55 +80 + ∠C = 180   {Angle  sum property}

135 + ∠C = 180

∠C = 180 - 135

∠C = 45

The side opposite to the biggest angle is the longest side

Biggest angle is ∠B. So the longest side is AC

Smallest angle is ∠C. So, the shortest side is BA

Please help!! I have no idea how to do this

Answers

Answer:

i have also no idea but plz mark me as brainlist

What is the quotient of the synthetic division problem below, written in polynomial form?

-1| 1 7 15 9 7​

Answers

Answer:

Step-by-step explanation:

If we have 5 terms to the right of the -1 outside the box thing, that means that our original polynomial is a 4th degree (the degree is one lower than the number of terms in order to allow for the constant which has no variable). Set up the synthetic division as shown in the problem. The rule is to bring down the first term, the 1 to the right of the box, multiply it by the number outside, -1, and put that product up under the next number in line inside the box. Like this:

-1 |   1    7     15     9     7

          -1                        

      1  

Now add straight down, multiply the sum by -1 and put that product up under the next term in line, the 15:

-1 |   1     7     15     9     7

            -1     -6              

      1     6  

Then add straight down again, multiply the sum by -1 and put that product up under the next term in line, the 9:

-1 |     1     7     15     9     7

              -1     -6    -9      

        1     6     9

Then do the same thing. Add straight down, multiply the sum by -1 and put the product up under the next term in line, the 7:

-1 |     1     7     15     9     7

              -1     -6    -9     0

        1     6      9     0     7

This is the final result. The last number, the 7 is the remainder, but the rest of it makes up what we call the depressed polynomial and is a polynomial that is one degree less than the degree we started with. So this depressed polynomial is a 3rd degree. The numbers are the coefficients on the x terms:

[tex]1x^3+6x^2+9x+0R7[/tex] or more simply stated:

[tex]x^3+6x^2+9xR7[/tex]

How many index laws are there?

Answers

Answer:

There are three laws of indices.

What should be subtracted from thrice the rational number -8/3 to get
5/2?

Answers

Answer: -21/2 or -10.5

Step-by-step explanation:

Let x be the number that is going to be subtracted from the rational number

3 × (-8/3) - x = 5/2 ⇒ Write the equation

-8 - x = 5/2 ⇒ Do multiplication

-8 - x + x = 5/2 +x ⇒ Add x on both sides

-8 = 5/2 + x ⇒ Simplify

-8 - 5/2 = 5/2 + x - 5/2 ⇒ Subtract 5/2 on both sides

x = -21/2 = -10.5 ⇒ Simplify

Hope this helps!! :)

Please let me know if you have any questions

Samuel played a video game. He gained some points, lost 87 points, and
finished with less than 320 points, Which of the following inequality
represents the given situation?
A.) n-87 < 320
B.) N + 87 < 320
C.) N-87 = 320
D.) n +87= 320

Answers

Answer:

A

Step-by-step explanation:

The correct inequality that represents the given situation is: A.) n - 87 < 320.

How to explain the equation

The situation states that Samuel played a video game and gained some points. Let's represent the points he gained as 'n'.

Next, it is mentioned that he lost 87 points. To represent this in the inequality, we subtract 87 from the initial points gained, which gives us 'n - 87'.

Finally, the situation specifies that Samuel finished with less than 320 points. This means that the final score, represented by 'n - 87', is less than 320.

This inequality represents that the initial points gained by Samuel (n) minus 87 points is less than 320 points, indicating that his final score is less than 320 points after losing 87 points.

Learn more about equations

https://brainly.com/question/2972832

#SPJ2

F={(-1, 2), (3, 2), (4,2), (0, 2)} Is Fa function and why/why not?

Answers

Answer:

yes it is a function         A

Step-by-step explanation:

Other Questions
MY NOTES Verify that the function satisfies the three hypotheses of Rolle's Theorem on the given interval. Then find all numbers c that satisfy the conclusion of Rolle's Theorem. (Enter your answers as a comma-separated list.) f(x) = 2x2 4x + 3, [1, 3 What is the simplified form of the following expression? Assume x > 0.32x16x2x4/24x2x4/244316x4124 1) The person was really helpful. They spoke to him.( join the sentence with appropriate relative terms)2) You Feel Bored. (make a wish)3) I am sure she has a good party dress ( make a sentence with may/might/must)4) The boy look quite aggressive. ( transitive verb, intransitive verb, linking verb)5) They .............(live) in the same house since they came to town. 2. help/ people/ a device/ communicate/ through/ brain waves/ emotions/. Find the measure of the indicated angle. For whom is this passage most likely intended? TotalWinnings6Translate I repeat the words into Spanish by filling in the missing letters:Yo retoaabrI repeat the words Venn diagrams: unions, intersections, and complementsAttached is the photo reference Where ____?have you beenare you beenhas you beenhave you werecual es lacorrecta? The volume of a prism with side lengths measured in millimeters is 20. How could this measurement be written? Check all that apply.20 millimeters20 mm320 mm220 square millimeters20 cubic millimeters Kenya solved the equation below. Negative 6 (x minus 2) + 3 x = negative 3 (x + 3) + 21 What is the solution to Kenya's equation? 4 12 no solution infinitely many solutions A company's stock price flucated over a period of four days. The table shows the change in stock price per day. The net change in the company's stock price over the four days Question on verbs in French Find the area of the triangle with vertices (0,0,0),(4,1,2), and (4,2,3). Ayer,yo al supermercado con mi mama You measure 49 turtles' weights, and find they have a mean weight of 80 ounces. Assume the population standard deviation is 6.1 ounces. Based on this, construct a 99% confidence interval for the true population mean turtle weight. Round your answers to 2 decimal places. Hey. I just need some advice. So I suck at math right like I dont know anything, or I should say I dont remember anything! So a little bit over the summer I have been doing Khan acadamey and what not. But I still cant seem to do anything right! So school here starts september 1st and Im gonna be a freshman in highschool. What do I do!!! Question 8 If f (2) = (1 + 3) and g (2) VO+ 7, find g (f (x)). 9(f()) = 1 + 10 O g(f ()) = VI + 3 +7 Og(f (x)) = v= + 10 Og(f (2)) = 2? + 10 What do you understand by moment of inertia and torque?Word limit 50-60Please don't copy from any sources. You can rewrite. Plagiarism will be check. Thank you. he came late into yes/no question